Регистрирайте сеРегистрирайте се

3-ти кръг, 1970 година


 
   Форум за математика Форуми -> Олимпиади и състезания за 9-12 клас
Предишната тема :: Следващата тема  
Автор Съобщение
blqaa
Начинаещ


Регистриран на: 29 Mar 2008
Мнения: 57

Репутация: 6.4Репутация: 6.4Репутация: 6.4Репутация: 6.4Репутация: 6.4Репутация: 6.4
гласове: 4

МнениеПуснато на: Wed Feb 11, 2009 10:04 pm    Заглавие: 3-ти кръг, 1970 година

Докажете, че [tex]\frac{1-a}{1+a}+\frac{1-b}{1+b}+\frac{1-b}{1+b}\ge \frac{3}{2}[/tex] за неотрицателни числа a, b и c с условието a+b+c=1.
Върнете се в началото
Вижте профила на потребителя Изпратете лично съобщение
Реклама







Пуснато на:     Заглавие: Реклама

Върнете се в началото
dim
Напреднал


Регистриран на: 28 Jul 2008
Мнения: 324

Репутация: 45.7Репутация: 45.7Репутация: 45.7Репутация: 45.7Репутация: 45.7
гласове: 21

МнениеПуснато на: Thu Feb 12, 2009 3:13 am    Заглавие:

Даденото неравенство е еквивалентно на:

[tex]\frac{3-3abc-(ab+bc+ca)+a+b+c}{(1+a)(1+b)(1+c)}\ge \frac{3}{2}[/tex]<=>[tex]\frac{4-3abc-(ab+bc+ca)}{(1+a)(1+b)(1+c)}\ge \frac{3}{2}[/tex]. Като поопростим стигаме до [tex]2-9abc-5(ab+bc+ca)\ge 0[/tex], но [tex]ab+bc+ca\le \frac{(a+b+c)^2}{3}=\frac{1}{3}=>2-9abc-\frac{5}{3}\ge 0[/tex]<=>[tex]abc\le \frac{1}{27}[/tex], което е вярно, защото най-голяма стойност изразът вляво достига при [tex]a=b=c=\frac{1}{3}[/tex](сумата [tex]a+b+c=1[/tex]-фиксирана).
Върнете се в началото
Вижте профила на потребителя Изпратете лично съобщение
blqaa
Начинаещ


Регистриран на: 29 Mar 2008
Мнения: 57

Репутация: 6.4Репутация: 6.4Репутация: 6.4Репутация: 6.4Репутация: 6.4Репутация: 6.4
гласове: 4

МнениеПуснато на: Thu Feb 12, 2009 10:26 am    Заглавие:

Други решения? Има много по елегантно.
Върнете се в началото
Вижте профила на потребителя Изпратете лично съобщение
JusTok
Редовен


Регистриран на: 26 Jul 2007
Мнения: 117
Местожителство: Варна
Репутация: 45.3Репутация: 45.3Репутация: 45.3Репутация: 45.3Репутация: 45.3
гласове: 24

МнениеПуснато на: Thu Feb 12, 2009 11:41 am    Заглавие:

[tex]\sum_{cyc}^{} \frac{1-a}{1+a}=\sum_{cyc}^{} \frac{1}{1+a} - (\sum_{cyc}^{} \frac{a}{1+a})=2\sum_{cyc}^{} \frac{1}{1+a}-3\ge 2.\frac{9}{4}-3=\frac{3}{2}[/tex]
равенство при a=b=c;
Върнете се в началото
Вижте профила на потребителя Изпратете лично съобщение
blqaa
Начинаещ


Регистриран на: 29 Mar 2008
Мнения: 57

Репутация: 6.4Репутация: 6.4Репутация: 6.4Репутация: 6.4Репутация: 6.4Репутация: 6.4
гласове: 4

МнениеПуснато на: Thu Feb 12, 2009 12:11 pm    Заглавие:

Браво. Аз използвам КБШ или "хубавото неравенство" още в началото и след това лесно излиза.
Върнете се в началото
Вижте профила на потребителя Изпратете лично съобщение
Покажи мнения от преди:   
   Форум за математика Форуми -> Олимпиади и състезания за 9-12 клас Часовете са според зоната GMT + 2 Часа
Страница 1 от 1

 
Идете на:  
Не Можете да пускате нови теми
Не Можете да отговаряте на темите
Не Можете да променяте съобщенията си
Не Можете да изтривате съобщенията си
Не Можете да гласувате в анкети
Може да прикачвате файлове
Може да сваляте файлове от този форум
Copyright © 2005-2021 math10.com.